Q12

 
cool2daysjh11
Thanks Received: 0
Vinny Gambini
Vinny Gambini
 
Posts: 1
Joined: May 18th, 2016
 
 
 

Q12

by cool2daysjh11 Thu May 26, 2016 7:24 pm

Hello.
I am not sure about Q12,13,14,17
I got right for three of them, but I just pick between two contenders (without firm reasons)

for Q12, (B) and (D)
Q13 (A) and (E)
Q14 (B) and (E)
Q17 (A) and (E)

Since there are not many conditions to restrict variables,
I stuck in the middle of too many options.
Did I miss some condition?
Please help me.
(I checked diagram, but my diagram is all the same with the post!)
User avatar
 
ohthatpatrick
Thanks Received: 3806
Atticus Finch
Atticus Finch
 
Posts: 4661
Joined: April 01st, 2011
 
 
 

Re: Q12

by ohthatpatrick Thu Jun 02, 2016 2:17 pm

When in doubt, write it out and check the rules.

For all those question for which you were down to two answer, just force yourself to write out complete scenarios and check them against the rules. They can't both be right! :)

There's not a ton to get from the setup.

8 things, 6 in and 2 out.

__ __ | __ __ | __ __ | __ __
....1...........2........3.......out

The most important rule is probably the last one, because it involves a chunk (chunks are always the most important part of a game).

If P is 1, there's an FH chunk on day 2.
If P is 2, there's an FH chunk on day 3.
Otherwise, P is 3 or Out.

For Q12, you should be taking one rule at a time and checking to see if any answers break that rule.

(C) breaks Rule 1
(D) breaks Rule 2
(A) breaks Rule 3
(E) breaks Rule 4

The correct answer is (B).

(Since you were considering (D), make sure you see why it's wrong. Rule 2 says that you can't put S or W in the afternoon. (D) puts W in the afternoon.)

The way our forum works, we don't wanna put explanations for 13, 14, and 17 on a thread for Q12.

But if you post a New Topic for each one of those, we'll be happy to answer them!

(sneak preview of Q13, in case this is enough to get you going for the rest of the Q's .....

13. __ __ | __ L | __ F | __ __

Any rules for L or F?

There's a rule that says L can't share a day with O or P. So neither O nor P can be in day 2.

Since O is ONLY allowed to be in day 2 or otherwise OUT, O is out.

__ __ | __ L | __ F | __ O

P can be 1, 3, or OUT.

We mentioned before that the P rule is the most important.
If P is 1, then FH is day 2. Is that possible?

No, because F is already locked into day 3. So we know that P cannot be day 1.

P is down to day 3 or OUT.

We've checked rules for L and F. But the question also involves "afternoons" and day 2 / day 3, so we want to check rules for that as well.

S and W cannot go in the afternoon. That's not super helpful. We still have 3 morning spots open, so S and W have options.

Who's left?
N and H, and they're essentially floaters (H is in a rule, but we've already decided P is 3 or OUT).

We should probably just try framing P in 3 vs. P being OUT.

__ __ | __ L | P F | __ O

__ __ | __ L | __ F | P O

In the second frame, the out column is full, so S, W, N, and H are all left.

S and W are forced to use a morning spot, but N and H can go anywhere.

Since the question is asking about Day 1's afternoon, it could be N or H.

(A) is correct

Notice that we could have already eliminated all the other answers
(B) O can never go on day 1
(D) and (E) S and W can never be an afternoon
(C) P can't be in day 1, because that would force FH to be day 2.

So this question is much more easily solved by focusing on reasons why the other 4 CANNOT be scheduled.

Since you were considering (E) for this one, which violates rule 2, it looks like you may have misunderstood rule 2. It was the difference maker in Q12 and Q13.
 
CrackLSAT170
Thanks Received: 0
Vinny Gambini
Vinny Gambini
 
Posts: 2
Joined: November 15th, 2015
 
 
 

Re: Q12

by CrackLSAT170 Mon Mar 13, 2017 8:57 am

Hi,

I am really confused about the CHOICE E of Q12. From the explanation, it can be eliminated with the rule 4, while I did not see any relationship between them. Could you please explain it? Many thanks.
User avatar
 
ohthatpatrick
Thanks Received: 3806
Atticus Finch
Atticus Finch
 
Posts: 4661
Joined: April 01st, 2011
 
 
 

Re: Q12

by ohthatpatrick Thu Mar 16, 2017 2:14 pm

Rule 4 gets triggered whenever pastels is on day 1 or 2.

In choice (E), pastels is the morning of day 2.

We've triggered the rule --- what is the 2nd half of it (the outcome) that we need to verify?

It says that lectures on the next day, which is day 3 since pastels was on day 2, must be fresco and history.

In (E), the lectures on day 3 are fresco and naturalism.